Prove that $(a - 1)^3 + (b - 1)^3 + (c - 1)^3 ge -dfrac{3}{4}$ where $a+b+c=3$prove inequalityProve that...

What Exploit Are These User Agents Trying to Use?

How to travel to Japan while expressing milk?

What's the meaning of "Sollensaussagen"?

Is it a bad idea to plug the other end of ESD strap to wall ground?

How does a dynamic QR code work?

Is it possible to create a QR code using text?

Is there an expression that means doing something right before you will need it rather than doing it in case you might need it?

Why was Sir Cadogan fired?

Why was the shrink from 8″ made only to 5.25″ and not smaller (4″ or less)

What reasons are there for a Capitalist to oppose a 100% inheritance tax?

Getting extremely large arrows with tikzcd

How can saying a song's name be a copyright violation?

How much mains leakage does an Ethernet connection to a PC induce, and what is the operating leakage path?

How dangerous is XSS

Why is the sentence "Das ist eine Nase" correct?

Can compressed videos be decoded back to their uncompresed original format?

Theorists sure want true answers to this!

My ex-girlfriend uses my Apple ID to log in to her iPad. Do I have to give her my Apple ID password to reset it?

Could the museum Saturn V's be refitted for one more flight?

Rotate ASCII Art by 45 Degrees

Can I hook these wires up to find the connection to a dead outlet?

Where would I need my direct neural interface to be implanted?

Processor speed limited at 0.4 Ghz

Car headlights in a world without electricity



Prove that $(a - 1)^3 + (b - 1)^3 + (c - 1)^3 ge -dfrac{3}{4}$ where $a+b+c=3$


prove inequalityProve that nearly all positive integers are equal to $a + b + c$ where $a | b$ and $b | c$, $a lt b lt c$If $a$ and $b$ are positive real numbers such that $a+b=1$, prove that $(a+1/a)^2+(b+1/b)^2ge 25/2$Prove that $a^acdot b^b>bigg(dfrac{a+b}{2}bigg)^{a+b}$ where $ane b$How find this range of the fucntion $f(a,b)=left(frac{1}{a}+frac{1}{b}+1right)(a-3b+15)$How prove this inequality $sumlimits_{cyc}frac{a^3}{b+c+d}ge dfrac{1}{3}$,if $sumlimits_{cyc}asqrt{bc}ge 1$How prove Reversing the Arithmetic mean – Geometric mean inequality?Proving or disproving an inequality regarding non-negative real numbers.$L^1$ inequality between ordered real numbers implies $L^2$ norm inequalityShow that $frac{1}{cosh(x)} + logleft(frac{cosh(x)}{1+cosh(x)}right) ge 0$













3












$begingroup$



If $a$, $b$, $c$ are non-negative numbers such that $a + b + c = 3$ then prove
$$(a - 1)^3 + (b - 1)^3 + (c - 1)^3 ge -frac{3}{4}$$




Here's what I did.



Let $c ge a ge b$.



We have that
begin{align*}
(c - 1)^3 + (a - 1)^3 &= (c + a - 2)(c^2 + a^2 - ca - c - a + 1)\
&ge (1 - b)left[dfrac{3}{4}(c + a)^2 - b + 4right]\
&=(1 - b)left[dfrac{3}{4}(3 - b)^2 - b + 4right]\
&= dfrac{1}{4}(1 - b)(3b^2 - 22b + 43)
end{align*}



That means



begin{align*}
(a - 1)^3 + (b - 1)^3 + (c - 1)^3 &ge dfrac{1}{4}(1 - b)(3b^2 - 22b + 43) + (b - 1)^3\
&= dfrac{1}{4}(b - 1)(b^2 + 14b - 39)
end{align*}



Since $c ge a ge b implies b le dfrac{a + b + c}{3} = 1$.



And this is where I am stuck right now.










share|cite|improve this question











$endgroup$












  • $begingroup$
    Your body and title are stating two different inequalities. Further please avoid titls only containing MathJax Expressions.
    $endgroup$
    – mrtaurho
    Mar 18 at 14:29










  • $begingroup$
    Could you please explain why?
    $endgroup$
    – Lê Thành Đạt
    Mar 18 at 14:32










  • $begingroup$
    Take a look at this Meta thread for instance. For myself I hate MathJax-only titles as I cannot open them within a new tab.
    $endgroup$
    – mrtaurho
    Mar 18 at 14:35


















3












$begingroup$



If $a$, $b$, $c$ are non-negative numbers such that $a + b + c = 3$ then prove
$$(a - 1)^3 + (b - 1)^3 + (c - 1)^3 ge -frac{3}{4}$$




Here's what I did.



Let $c ge a ge b$.



We have that
begin{align*}
(c - 1)^3 + (a - 1)^3 &= (c + a - 2)(c^2 + a^2 - ca - c - a + 1)\
&ge (1 - b)left[dfrac{3}{4}(c + a)^2 - b + 4right]\
&=(1 - b)left[dfrac{3}{4}(3 - b)^2 - b + 4right]\
&= dfrac{1}{4}(1 - b)(3b^2 - 22b + 43)
end{align*}



That means



begin{align*}
(a - 1)^3 + (b - 1)^3 + (c - 1)^3 &ge dfrac{1}{4}(1 - b)(3b^2 - 22b + 43) + (b - 1)^3\
&= dfrac{1}{4}(b - 1)(b^2 + 14b - 39)
end{align*}



Since $c ge a ge b implies b le dfrac{a + b + c}{3} = 1$.



And this is where I am stuck right now.










share|cite|improve this question











$endgroup$












  • $begingroup$
    Your body and title are stating two different inequalities. Further please avoid titls only containing MathJax Expressions.
    $endgroup$
    – mrtaurho
    Mar 18 at 14:29










  • $begingroup$
    Could you please explain why?
    $endgroup$
    – Lê Thành Đạt
    Mar 18 at 14:32










  • $begingroup$
    Take a look at this Meta thread for instance. For myself I hate MathJax-only titles as I cannot open them within a new tab.
    $endgroup$
    – mrtaurho
    Mar 18 at 14:35
















3












3








3


1



$begingroup$



If $a$, $b$, $c$ are non-negative numbers such that $a + b + c = 3$ then prove
$$(a - 1)^3 + (b - 1)^3 + (c - 1)^3 ge -frac{3}{4}$$




Here's what I did.



Let $c ge a ge b$.



We have that
begin{align*}
(c - 1)^3 + (a - 1)^3 &= (c + a - 2)(c^2 + a^2 - ca - c - a + 1)\
&ge (1 - b)left[dfrac{3}{4}(c + a)^2 - b + 4right]\
&=(1 - b)left[dfrac{3}{4}(3 - b)^2 - b + 4right]\
&= dfrac{1}{4}(1 - b)(3b^2 - 22b + 43)
end{align*}



That means



begin{align*}
(a - 1)^3 + (b - 1)^3 + (c - 1)^3 &ge dfrac{1}{4}(1 - b)(3b^2 - 22b + 43) + (b - 1)^3\
&= dfrac{1}{4}(b - 1)(b^2 + 14b - 39)
end{align*}



Since $c ge a ge b implies b le dfrac{a + b + c}{3} = 1$.



And this is where I am stuck right now.










share|cite|improve this question











$endgroup$





If $a$, $b$, $c$ are non-negative numbers such that $a + b + c = 3$ then prove
$$(a - 1)^3 + (b - 1)^3 + (c - 1)^3 ge -frac{3}{4}$$




Here's what I did.



Let $c ge a ge b$.



We have that
begin{align*}
(c - 1)^3 + (a - 1)^3 &= (c + a - 2)(c^2 + a^2 - ca - c - a + 1)\
&ge (1 - b)left[dfrac{3}{4}(c + a)^2 - b + 4right]\
&=(1 - b)left[dfrac{3}{4}(3 - b)^2 - b + 4right]\
&= dfrac{1}{4}(1 - b)(3b^2 - 22b + 43)
end{align*}



That means



begin{align*}
(a - 1)^3 + (b - 1)^3 + (c - 1)^3 &ge dfrac{1}{4}(1 - b)(3b^2 - 22b + 43) + (b - 1)^3\
&= dfrac{1}{4}(b - 1)(b^2 + 14b - 39)
end{align*}



Since $c ge a ge b implies b le dfrac{a + b + c}{3} = 1$.



And this is where I am stuck right now.







inequality tangent-line-method






share|cite|improve this question















share|cite|improve this question













share|cite|improve this question




share|cite|improve this question








edited Mar 18 at 18:49









Maria Mazur

49.5k1361124




49.5k1361124










asked Mar 18 at 14:27









Lê Thành ĐạtLê Thành Đạt

35813




35813












  • $begingroup$
    Your body and title are stating two different inequalities. Further please avoid titls only containing MathJax Expressions.
    $endgroup$
    – mrtaurho
    Mar 18 at 14:29










  • $begingroup$
    Could you please explain why?
    $endgroup$
    – Lê Thành Đạt
    Mar 18 at 14:32










  • $begingroup$
    Take a look at this Meta thread for instance. For myself I hate MathJax-only titles as I cannot open them within a new tab.
    $endgroup$
    – mrtaurho
    Mar 18 at 14:35




















  • $begingroup$
    Your body and title are stating two different inequalities. Further please avoid titls only containing MathJax Expressions.
    $endgroup$
    – mrtaurho
    Mar 18 at 14:29










  • $begingroup$
    Could you please explain why?
    $endgroup$
    – Lê Thành Đạt
    Mar 18 at 14:32










  • $begingroup$
    Take a look at this Meta thread for instance. For myself I hate MathJax-only titles as I cannot open them within a new tab.
    $endgroup$
    – mrtaurho
    Mar 18 at 14:35


















$begingroup$
Your body and title are stating two different inequalities. Further please avoid titls only containing MathJax Expressions.
$endgroup$
– mrtaurho
Mar 18 at 14:29




$begingroup$
Your body and title are stating two different inequalities. Further please avoid titls only containing MathJax Expressions.
$endgroup$
– mrtaurho
Mar 18 at 14:29












$begingroup$
Could you please explain why?
$endgroup$
– Lê Thành Đạt
Mar 18 at 14:32




$begingroup$
Could you please explain why?
$endgroup$
– Lê Thành Đạt
Mar 18 at 14:32












$begingroup$
Take a look at this Meta thread for instance. For myself I hate MathJax-only titles as I cannot open them within a new tab.
$endgroup$
– mrtaurho
Mar 18 at 14:35






$begingroup$
Take a look at this Meta thread for instance. For myself I hate MathJax-only titles as I cannot open them within a new tab.
$endgroup$
– mrtaurho
Mar 18 at 14:35












5 Answers
5






active

oldest

votes


















4












$begingroup$

You can prove that $$color{red}{(x-1)^3geq {3over 4}x-1}$$ for all nonegative $x$; it is equivalent to $x(2x-3)^2ge 0$, so $$(a-1)^3+(b-1)^3+(c-1)^3geq {3over 4}(a+b+c)-3= -{3over 4}$$






share|cite|improve this answer











$endgroup$









  • 2




    $begingroup$
    I think this is the simplest approach...[+1]!
    $endgroup$
    – Dr. Mathva
    Mar 18 at 18:43



















2












$begingroup$

Perhaps a different approach than you are using. Using the Lagrange multipliers, we define the auxiliary problem
$$ F = (x-1)^3 + (y-1)^3 + (z-1)^3 - lambda(x + y + z - 3). $$
Differentiating and setting to $0$ we obtain the conditions
begin{align}
3(x-1)^2 = lambda \
3(y-1)^2 = lambda \
3(z-1)^2 = lambda
end{align}

It is easy to see that if we set $x,y,z$ to the critical point obtained by these conditions we will reach a contradiction with the constraint $x + y + z = 3$ (check this yourself). Then, it must be that the extremum if it exists must have at least one variable (say $z$) which is $0$. This leads to a value of $lambda = 3/4$ and finally, one can show that any permutation of the triplet $(3/2,3/2,0)$ leads to the minimum value. You should also check that this is indeed the minimum.






share|cite|improve this answer









$endgroup$













  • $begingroup$
    I wish I could go with your solution. But I haven't studied the Lagrange multipliers yet. (I self-thought that at home but I can't use it at school.) So do you have any other solution for me?
    $endgroup$
    – Lê Thành Đạt
    Mar 18 at 15:02



















1












$begingroup$

My sketchy proof:



First you need to prove that if $x+y+z=0$, then $x^3 + y^3 + z^3 = 3xyz$



Since $a+b+c=3$, we have $(a - 1)+(b-1)+(c-1)=0$, therefore, $S = (a - 1)^3+(b-1)^3+(c-1)^3=3(a - 1)(b-1)(c-1)$



Replace $c = 3 - a -b$, we have $S = 3(a-1)(b-1)(2-a-b)$



Expand this, we have $S/3 = a^2 + b^2 + 4ab - 3(a+b) - ab(a+b) + 2$



We can safely assume that $a geq b$, since the order $a$, $b$ doesn't matter. So we have: $(a-b)^2 geq 0$ or $a^2 + b^2 geq 2ab$. Equality happens when $a = b$.



So $S/3 geq 6ab - 3(a+b) - ab(a+b) + 2$



Since $c geq 0$, we have $3 - a - b geq 0$, or $-(a+b) geq -3$. Equality happens when $a + b = 3$.



So $S/3 geq 6ab + 3(-3) + ab(-3) + 2 = 3ab - 7$, or $S geq 9ab - 21$. Equality happens when $a = b$ and $a + b = 3$, or $a = b = 3/2$. Replace $a$, $b$ in $9ab-21$ with $a = b = 3/2$ we have $S geq -3/4$



I'm not satisfy with the last step, though.






share|cite|improve this answer









$endgroup$













  • $begingroup$
    That first step is genius. I would never have thought of that. Also, you can shorten the second last and last steps.
    $endgroup$
    – Lê Thành Đạt
    Mar 18 at 16:05






  • 1




    $begingroup$
    The last step isn’t correct, unfortunately. There is no reason to believe $9ab-21$ achieves minimum when $a=b=frac32$
    $endgroup$
    – Macavity
    Mar 19 at 2:15





















0












$begingroup$

Hope it will help!



WLOG $ bge age cRightarrow cle 1Rightarrow 1-cge 0$



We have: $$fleft(a,b,cright)=left(a-1right)^3+left(b-1right)^3+left(c-1right)^3$$



And $$fleft(frac{a+b}{2};frac{a+b}{2};cright)=2left(frac{a+b}{2}-1right)^3+left(c-1right)^3$$



Note that:$$fleft(a,b,cright)-fleft(frac{a+b}{2};frac{a+b}{2};cright)=frac{3}{4}left(a-bright)^2left(a+b-2right)=frac{3}{4}left(a-bright)^2left(1-cright)ge 0$$



So you need to prove this inequality in the case $a=b$ the condition gives $c=3-2a$



Thus we need to prove $$2left(a-1right)^3+left(3-2a-1right)^3ge -frac{3}{4}$$



Or $$-frac{3}{4}left(2a-3right)left(4a^2-6a+3right)ge 0$$






share|cite|improve this answer











$endgroup$





















    0












    $begingroup$

    The homogenisation gives
    $$sum_{cyc}(a^3-a^2b-a^2c+2abc)geq0,$$
    which is true by Schur.



    Indeed, we need to prove that
    $$sum_{cyc}(a-1)^3+frac{3}{4}geq0$$ or
    $$sum_{cyc}left(a^3-3a^2+3-1+frac{1}{4}right)geq0$$ or
    $$sum_{cyc}(a^3-(a+b+c)a^2)+frac{27}{4}$$ or
    $$4sum_{cyc}(-a^2b-a^2c)+(a+b+c)^3geq0$$ or
    $$sum_{cyc}(-4a^2b-4a^2c+a^3+3a^2b+3a^2c+2abc)geq0$$ or
    $$sum_{cyc}(a^3-a^2b-a^2c+2abc)geq0,$$ which is obvious.






    share|cite|improve this answer











    $endgroup$













    • $begingroup$
      Could you please explain more?
      $endgroup$
      – Lê Thành Đạt
      Mar 18 at 15:14










    • $begingroup$
      Yes, of course! I am driving now. I explain it later
      $endgroup$
      – Michael Rozenberg
      Mar 18 at 15:35










    • $begingroup$
      I can wait. Drive carefully!
      $endgroup$
      – Lê Thành Đạt
      Mar 18 at 15:37










    • $begingroup$
      It is just $9(a-1)^3+9(b-1)^3+9(c-1)^3ge -frac{left(a+b+cright)^3}{4}$.
      $endgroup$
      – Word Shallow
      Mar 18 at 16:35












    Your Answer





    StackExchange.ifUsing("editor", function () {
    return StackExchange.using("mathjaxEditing", function () {
    StackExchange.MarkdownEditor.creationCallbacks.add(function (editor, postfix) {
    StackExchange.mathjaxEditing.prepareWmdForMathJax(editor, postfix, [["$", "$"], ["\\(","\\)"]]);
    });
    });
    }, "mathjax-editing");

    StackExchange.ready(function() {
    var channelOptions = {
    tags: "".split(" "),
    id: "69"
    };
    initTagRenderer("".split(" "), "".split(" "), channelOptions);

    StackExchange.using("externalEditor", function() {
    // Have to fire editor after snippets, if snippets enabled
    if (StackExchange.settings.snippets.snippetsEnabled) {
    StackExchange.using("snippets", function() {
    createEditor();
    });
    }
    else {
    createEditor();
    }
    });

    function createEditor() {
    StackExchange.prepareEditor({
    heartbeatType: 'answer',
    autoActivateHeartbeat: false,
    convertImagesToLinks: true,
    noModals: true,
    showLowRepImageUploadWarning: true,
    reputationToPostImages: 10,
    bindNavPrevention: true,
    postfix: "",
    imageUploader: {
    brandingHtml: "Powered by u003ca class="icon-imgur-white" href="https://imgur.com/"u003eu003c/au003e",
    contentPolicyHtml: "User contributions licensed under u003ca href="https://creativecommons.org/licenses/by-sa/3.0/"u003ecc by-sa 3.0 with attribution requiredu003c/au003e u003ca href="https://stackoverflow.com/legal/content-policy"u003e(content policy)u003c/au003e",
    allowUrls: true
    },
    noCode: true, onDemand: true,
    discardSelector: ".discard-answer"
    ,immediatelyShowMarkdownHelp:true
    });


    }
    });














    draft saved

    draft discarded


















    StackExchange.ready(
    function () {
    StackExchange.openid.initPostLogin('.new-post-login', 'https%3a%2f%2fmath.stackexchange.com%2fquestions%2f3152853%2fprove-that-a-13-b-13-c-13-ge-dfrac34-where-abc-3%23new-answer', 'question_page');
    }
    );

    Post as a guest















    Required, but never shown

























    5 Answers
    5






    active

    oldest

    votes








    5 Answers
    5






    active

    oldest

    votes









    active

    oldest

    votes






    active

    oldest

    votes









    4












    $begingroup$

    You can prove that $$color{red}{(x-1)^3geq {3over 4}x-1}$$ for all nonegative $x$; it is equivalent to $x(2x-3)^2ge 0$, so $$(a-1)^3+(b-1)^3+(c-1)^3geq {3over 4}(a+b+c)-3= -{3over 4}$$






    share|cite|improve this answer











    $endgroup$









    • 2




      $begingroup$
      I think this is the simplest approach...[+1]!
      $endgroup$
      – Dr. Mathva
      Mar 18 at 18:43
















    4












    $begingroup$

    You can prove that $$color{red}{(x-1)^3geq {3over 4}x-1}$$ for all nonegative $x$; it is equivalent to $x(2x-3)^2ge 0$, so $$(a-1)^3+(b-1)^3+(c-1)^3geq {3over 4}(a+b+c)-3= -{3over 4}$$






    share|cite|improve this answer











    $endgroup$









    • 2




      $begingroup$
      I think this is the simplest approach...[+1]!
      $endgroup$
      – Dr. Mathva
      Mar 18 at 18:43














    4












    4








    4





    $begingroup$

    You can prove that $$color{red}{(x-1)^3geq {3over 4}x-1}$$ for all nonegative $x$; it is equivalent to $x(2x-3)^2ge 0$, so $$(a-1)^3+(b-1)^3+(c-1)^3geq {3over 4}(a+b+c)-3= -{3over 4}$$






    share|cite|improve this answer











    $endgroup$



    You can prove that $$color{red}{(x-1)^3geq {3over 4}x-1}$$ for all nonegative $x$; it is equivalent to $x(2x-3)^2ge 0$, so $$(a-1)^3+(b-1)^3+(c-1)^3geq {3over 4}(a+b+c)-3= -{3over 4}$$







    share|cite|improve this answer














    share|cite|improve this answer



    share|cite|improve this answer








    edited Mar 18 at 18:11

























    answered Mar 18 at 18:05









    Maria MazurMaria Mazur

    49.5k1361124




    49.5k1361124








    • 2




      $begingroup$
      I think this is the simplest approach...[+1]!
      $endgroup$
      – Dr. Mathva
      Mar 18 at 18:43














    • 2




      $begingroup$
      I think this is the simplest approach...[+1]!
      $endgroup$
      – Dr. Mathva
      Mar 18 at 18:43








    2




    2




    $begingroup$
    I think this is the simplest approach...[+1]!
    $endgroup$
    – Dr. Mathva
    Mar 18 at 18:43




    $begingroup$
    I think this is the simplest approach...[+1]!
    $endgroup$
    – Dr. Mathva
    Mar 18 at 18:43











    2












    $begingroup$

    Perhaps a different approach than you are using. Using the Lagrange multipliers, we define the auxiliary problem
    $$ F = (x-1)^3 + (y-1)^3 + (z-1)^3 - lambda(x + y + z - 3). $$
    Differentiating and setting to $0$ we obtain the conditions
    begin{align}
    3(x-1)^2 = lambda \
    3(y-1)^2 = lambda \
    3(z-1)^2 = lambda
    end{align}

    It is easy to see that if we set $x,y,z$ to the critical point obtained by these conditions we will reach a contradiction with the constraint $x + y + z = 3$ (check this yourself). Then, it must be that the extremum if it exists must have at least one variable (say $z$) which is $0$. This leads to a value of $lambda = 3/4$ and finally, one can show that any permutation of the triplet $(3/2,3/2,0)$ leads to the minimum value. You should also check that this is indeed the minimum.






    share|cite|improve this answer









    $endgroup$













    • $begingroup$
      I wish I could go with your solution. But I haven't studied the Lagrange multipliers yet. (I self-thought that at home but I can't use it at school.) So do you have any other solution for me?
      $endgroup$
      – Lê Thành Đạt
      Mar 18 at 15:02
















    2












    $begingroup$

    Perhaps a different approach than you are using. Using the Lagrange multipliers, we define the auxiliary problem
    $$ F = (x-1)^3 + (y-1)^3 + (z-1)^3 - lambda(x + y + z - 3). $$
    Differentiating and setting to $0$ we obtain the conditions
    begin{align}
    3(x-1)^2 = lambda \
    3(y-1)^2 = lambda \
    3(z-1)^2 = lambda
    end{align}

    It is easy to see that if we set $x,y,z$ to the critical point obtained by these conditions we will reach a contradiction with the constraint $x + y + z = 3$ (check this yourself). Then, it must be that the extremum if it exists must have at least one variable (say $z$) which is $0$. This leads to a value of $lambda = 3/4$ and finally, one can show that any permutation of the triplet $(3/2,3/2,0)$ leads to the minimum value. You should also check that this is indeed the minimum.






    share|cite|improve this answer









    $endgroup$













    • $begingroup$
      I wish I could go with your solution. But I haven't studied the Lagrange multipliers yet. (I self-thought that at home but I can't use it at school.) So do you have any other solution for me?
      $endgroup$
      – Lê Thành Đạt
      Mar 18 at 15:02














    2












    2








    2





    $begingroup$

    Perhaps a different approach than you are using. Using the Lagrange multipliers, we define the auxiliary problem
    $$ F = (x-1)^3 + (y-1)^3 + (z-1)^3 - lambda(x + y + z - 3). $$
    Differentiating and setting to $0$ we obtain the conditions
    begin{align}
    3(x-1)^2 = lambda \
    3(y-1)^2 = lambda \
    3(z-1)^2 = lambda
    end{align}

    It is easy to see that if we set $x,y,z$ to the critical point obtained by these conditions we will reach a contradiction with the constraint $x + y + z = 3$ (check this yourself). Then, it must be that the extremum if it exists must have at least one variable (say $z$) which is $0$. This leads to a value of $lambda = 3/4$ and finally, one can show that any permutation of the triplet $(3/2,3/2,0)$ leads to the minimum value. You should also check that this is indeed the minimum.






    share|cite|improve this answer









    $endgroup$



    Perhaps a different approach than you are using. Using the Lagrange multipliers, we define the auxiliary problem
    $$ F = (x-1)^3 + (y-1)^3 + (z-1)^3 - lambda(x + y + z - 3). $$
    Differentiating and setting to $0$ we obtain the conditions
    begin{align}
    3(x-1)^2 = lambda \
    3(y-1)^2 = lambda \
    3(z-1)^2 = lambda
    end{align}

    It is easy to see that if we set $x,y,z$ to the critical point obtained by these conditions we will reach a contradiction with the constraint $x + y + z = 3$ (check this yourself). Then, it must be that the extremum if it exists must have at least one variable (say $z$) which is $0$. This leads to a value of $lambda = 3/4$ and finally, one can show that any permutation of the triplet $(3/2,3/2,0)$ leads to the minimum value. You should also check that this is indeed the minimum.







    share|cite|improve this answer












    share|cite|improve this answer



    share|cite|improve this answer










    answered Mar 18 at 14:59









    GregoryGregory

    1,346412




    1,346412












    • $begingroup$
      I wish I could go with your solution. But I haven't studied the Lagrange multipliers yet. (I self-thought that at home but I can't use it at school.) So do you have any other solution for me?
      $endgroup$
      – Lê Thành Đạt
      Mar 18 at 15:02


















    • $begingroup$
      I wish I could go with your solution. But I haven't studied the Lagrange multipliers yet. (I self-thought that at home but I can't use it at school.) So do you have any other solution for me?
      $endgroup$
      – Lê Thành Đạt
      Mar 18 at 15:02
















    $begingroup$
    I wish I could go with your solution. But I haven't studied the Lagrange multipliers yet. (I self-thought that at home but I can't use it at school.) So do you have any other solution for me?
    $endgroup$
    – Lê Thành Đạt
    Mar 18 at 15:02




    $begingroup$
    I wish I could go with your solution. But I haven't studied the Lagrange multipliers yet. (I self-thought that at home but I can't use it at school.) So do you have any other solution for me?
    $endgroup$
    – Lê Thành Đạt
    Mar 18 at 15:02











    1












    $begingroup$

    My sketchy proof:



    First you need to prove that if $x+y+z=0$, then $x^3 + y^3 + z^3 = 3xyz$



    Since $a+b+c=3$, we have $(a - 1)+(b-1)+(c-1)=0$, therefore, $S = (a - 1)^3+(b-1)^3+(c-1)^3=3(a - 1)(b-1)(c-1)$



    Replace $c = 3 - a -b$, we have $S = 3(a-1)(b-1)(2-a-b)$



    Expand this, we have $S/3 = a^2 + b^2 + 4ab - 3(a+b) - ab(a+b) + 2$



    We can safely assume that $a geq b$, since the order $a$, $b$ doesn't matter. So we have: $(a-b)^2 geq 0$ or $a^2 + b^2 geq 2ab$. Equality happens when $a = b$.



    So $S/3 geq 6ab - 3(a+b) - ab(a+b) + 2$



    Since $c geq 0$, we have $3 - a - b geq 0$, or $-(a+b) geq -3$. Equality happens when $a + b = 3$.



    So $S/3 geq 6ab + 3(-3) + ab(-3) + 2 = 3ab - 7$, or $S geq 9ab - 21$. Equality happens when $a = b$ and $a + b = 3$, or $a = b = 3/2$. Replace $a$, $b$ in $9ab-21$ with $a = b = 3/2$ we have $S geq -3/4$



    I'm not satisfy with the last step, though.






    share|cite|improve this answer









    $endgroup$













    • $begingroup$
      That first step is genius. I would never have thought of that. Also, you can shorten the second last and last steps.
      $endgroup$
      – Lê Thành Đạt
      Mar 18 at 16:05






    • 1




      $begingroup$
      The last step isn’t correct, unfortunately. There is no reason to believe $9ab-21$ achieves minimum when $a=b=frac32$
      $endgroup$
      – Macavity
      Mar 19 at 2:15


















    1












    $begingroup$

    My sketchy proof:



    First you need to prove that if $x+y+z=0$, then $x^3 + y^3 + z^3 = 3xyz$



    Since $a+b+c=3$, we have $(a - 1)+(b-1)+(c-1)=0$, therefore, $S = (a - 1)^3+(b-1)^3+(c-1)^3=3(a - 1)(b-1)(c-1)$



    Replace $c = 3 - a -b$, we have $S = 3(a-1)(b-1)(2-a-b)$



    Expand this, we have $S/3 = a^2 + b^2 + 4ab - 3(a+b) - ab(a+b) + 2$



    We can safely assume that $a geq b$, since the order $a$, $b$ doesn't matter. So we have: $(a-b)^2 geq 0$ or $a^2 + b^2 geq 2ab$. Equality happens when $a = b$.



    So $S/3 geq 6ab - 3(a+b) - ab(a+b) + 2$



    Since $c geq 0$, we have $3 - a - b geq 0$, or $-(a+b) geq -3$. Equality happens when $a + b = 3$.



    So $S/3 geq 6ab + 3(-3) + ab(-3) + 2 = 3ab - 7$, or $S geq 9ab - 21$. Equality happens when $a = b$ and $a + b = 3$, or $a = b = 3/2$. Replace $a$, $b$ in $9ab-21$ with $a = b = 3/2$ we have $S geq -3/4$



    I'm not satisfy with the last step, though.






    share|cite|improve this answer









    $endgroup$













    • $begingroup$
      That first step is genius. I would never have thought of that. Also, you can shorten the second last and last steps.
      $endgroup$
      – Lê Thành Đạt
      Mar 18 at 16:05






    • 1




      $begingroup$
      The last step isn’t correct, unfortunately. There is no reason to believe $9ab-21$ achieves minimum when $a=b=frac32$
      $endgroup$
      – Macavity
      Mar 19 at 2:15
















    1












    1








    1





    $begingroup$

    My sketchy proof:



    First you need to prove that if $x+y+z=0$, then $x^3 + y^3 + z^3 = 3xyz$



    Since $a+b+c=3$, we have $(a - 1)+(b-1)+(c-1)=0$, therefore, $S = (a - 1)^3+(b-1)^3+(c-1)^3=3(a - 1)(b-1)(c-1)$



    Replace $c = 3 - a -b$, we have $S = 3(a-1)(b-1)(2-a-b)$



    Expand this, we have $S/3 = a^2 + b^2 + 4ab - 3(a+b) - ab(a+b) + 2$



    We can safely assume that $a geq b$, since the order $a$, $b$ doesn't matter. So we have: $(a-b)^2 geq 0$ or $a^2 + b^2 geq 2ab$. Equality happens when $a = b$.



    So $S/3 geq 6ab - 3(a+b) - ab(a+b) + 2$



    Since $c geq 0$, we have $3 - a - b geq 0$, or $-(a+b) geq -3$. Equality happens when $a + b = 3$.



    So $S/3 geq 6ab + 3(-3) + ab(-3) + 2 = 3ab - 7$, or $S geq 9ab - 21$. Equality happens when $a = b$ and $a + b = 3$, or $a = b = 3/2$. Replace $a$, $b$ in $9ab-21$ with $a = b = 3/2$ we have $S geq -3/4$



    I'm not satisfy with the last step, though.






    share|cite|improve this answer









    $endgroup$



    My sketchy proof:



    First you need to prove that if $x+y+z=0$, then $x^3 + y^3 + z^3 = 3xyz$



    Since $a+b+c=3$, we have $(a - 1)+(b-1)+(c-1)=0$, therefore, $S = (a - 1)^3+(b-1)^3+(c-1)^3=3(a - 1)(b-1)(c-1)$



    Replace $c = 3 - a -b$, we have $S = 3(a-1)(b-1)(2-a-b)$



    Expand this, we have $S/3 = a^2 + b^2 + 4ab - 3(a+b) - ab(a+b) + 2$



    We can safely assume that $a geq b$, since the order $a$, $b$ doesn't matter. So we have: $(a-b)^2 geq 0$ or $a^2 + b^2 geq 2ab$. Equality happens when $a = b$.



    So $S/3 geq 6ab - 3(a+b) - ab(a+b) + 2$



    Since $c geq 0$, we have $3 - a - b geq 0$, or $-(a+b) geq -3$. Equality happens when $a + b = 3$.



    So $S/3 geq 6ab + 3(-3) + ab(-3) + 2 = 3ab - 7$, or $S geq 9ab - 21$. Equality happens when $a = b$ and $a + b = 3$, or $a = b = 3/2$. Replace $a$, $b$ in $9ab-21$ with $a = b = 3/2$ we have $S geq -3/4$



    I'm not satisfy with the last step, though.







    share|cite|improve this answer












    share|cite|improve this answer



    share|cite|improve this answer










    answered Mar 18 at 15:59









    tntxtnttntxtnt

    11914




    11914












    • $begingroup$
      That first step is genius. I would never have thought of that. Also, you can shorten the second last and last steps.
      $endgroup$
      – Lê Thành Đạt
      Mar 18 at 16:05






    • 1




      $begingroup$
      The last step isn’t correct, unfortunately. There is no reason to believe $9ab-21$ achieves minimum when $a=b=frac32$
      $endgroup$
      – Macavity
      Mar 19 at 2:15




















    • $begingroup$
      That first step is genius. I would never have thought of that. Also, you can shorten the second last and last steps.
      $endgroup$
      – Lê Thành Đạt
      Mar 18 at 16:05






    • 1




      $begingroup$
      The last step isn’t correct, unfortunately. There is no reason to believe $9ab-21$ achieves minimum when $a=b=frac32$
      $endgroup$
      – Macavity
      Mar 19 at 2:15


















    $begingroup$
    That first step is genius. I would never have thought of that. Also, you can shorten the second last and last steps.
    $endgroup$
    – Lê Thành Đạt
    Mar 18 at 16:05




    $begingroup$
    That first step is genius. I would never have thought of that. Also, you can shorten the second last and last steps.
    $endgroup$
    – Lê Thành Đạt
    Mar 18 at 16:05




    1




    1




    $begingroup$
    The last step isn’t correct, unfortunately. There is no reason to believe $9ab-21$ achieves minimum when $a=b=frac32$
    $endgroup$
    – Macavity
    Mar 19 at 2:15






    $begingroup$
    The last step isn’t correct, unfortunately. There is no reason to believe $9ab-21$ achieves minimum when $a=b=frac32$
    $endgroup$
    – Macavity
    Mar 19 at 2:15













    0












    $begingroup$

    Hope it will help!



    WLOG $ bge age cRightarrow cle 1Rightarrow 1-cge 0$



    We have: $$fleft(a,b,cright)=left(a-1right)^3+left(b-1right)^3+left(c-1right)^3$$



    And $$fleft(frac{a+b}{2};frac{a+b}{2};cright)=2left(frac{a+b}{2}-1right)^3+left(c-1right)^3$$



    Note that:$$fleft(a,b,cright)-fleft(frac{a+b}{2};frac{a+b}{2};cright)=frac{3}{4}left(a-bright)^2left(a+b-2right)=frac{3}{4}left(a-bright)^2left(1-cright)ge 0$$



    So you need to prove this inequality in the case $a=b$ the condition gives $c=3-2a$



    Thus we need to prove $$2left(a-1right)^3+left(3-2a-1right)^3ge -frac{3}{4}$$



    Or $$-frac{3}{4}left(2a-3right)left(4a^2-6a+3right)ge 0$$






    share|cite|improve this answer











    $endgroup$


















      0












      $begingroup$

      Hope it will help!



      WLOG $ bge age cRightarrow cle 1Rightarrow 1-cge 0$



      We have: $$fleft(a,b,cright)=left(a-1right)^3+left(b-1right)^3+left(c-1right)^3$$



      And $$fleft(frac{a+b}{2};frac{a+b}{2};cright)=2left(frac{a+b}{2}-1right)^3+left(c-1right)^3$$



      Note that:$$fleft(a,b,cright)-fleft(frac{a+b}{2};frac{a+b}{2};cright)=frac{3}{4}left(a-bright)^2left(a+b-2right)=frac{3}{4}left(a-bright)^2left(1-cright)ge 0$$



      So you need to prove this inequality in the case $a=b$ the condition gives $c=3-2a$



      Thus we need to prove $$2left(a-1right)^3+left(3-2a-1right)^3ge -frac{3}{4}$$



      Or $$-frac{3}{4}left(2a-3right)left(4a^2-6a+3right)ge 0$$






      share|cite|improve this answer











      $endgroup$
















        0












        0








        0





        $begingroup$

        Hope it will help!



        WLOG $ bge age cRightarrow cle 1Rightarrow 1-cge 0$



        We have: $$fleft(a,b,cright)=left(a-1right)^3+left(b-1right)^3+left(c-1right)^3$$



        And $$fleft(frac{a+b}{2};frac{a+b}{2};cright)=2left(frac{a+b}{2}-1right)^3+left(c-1right)^3$$



        Note that:$$fleft(a,b,cright)-fleft(frac{a+b}{2};frac{a+b}{2};cright)=frac{3}{4}left(a-bright)^2left(a+b-2right)=frac{3}{4}left(a-bright)^2left(1-cright)ge 0$$



        So you need to prove this inequality in the case $a=b$ the condition gives $c=3-2a$



        Thus we need to prove $$2left(a-1right)^3+left(3-2a-1right)^3ge -frac{3}{4}$$



        Or $$-frac{3}{4}left(2a-3right)left(4a^2-6a+3right)ge 0$$






        share|cite|improve this answer











        $endgroup$



        Hope it will help!



        WLOG $ bge age cRightarrow cle 1Rightarrow 1-cge 0$



        We have: $$fleft(a,b,cright)=left(a-1right)^3+left(b-1right)^3+left(c-1right)^3$$



        And $$fleft(frac{a+b}{2};frac{a+b}{2};cright)=2left(frac{a+b}{2}-1right)^3+left(c-1right)^3$$



        Note that:$$fleft(a,b,cright)-fleft(frac{a+b}{2};frac{a+b}{2};cright)=frac{3}{4}left(a-bright)^2left(a+b-2right)=frac{3}{4}left(a-bright)^2left(1-cright)ge 0$$



        So you need to prove this inequality in the case $a=b$ the condition gives $c=3-2a$



        Thus we need to prove $$2left(a-1right)^3+left(3-2a-1right)^3ge -frac{3}{4}$$



        Or $$-frac{3}{4}left(2a-3right)left(4a^2-6a+3right)ge 0$$







        share|cite|improve this answer














        share|cite|improve this answer



        share|cite|improve this answer








        edited Mar 18 at 16:30

























        answered Mar 18 at 16:23









        Word ShallowWord Shallow

        1,1832622




        1,1832622























            0












            $begingroup$

            The homogenisation gives
            $$sum_{cyc}(a^3-a^2b-a^2c+2abc)geq0,$$
            which is true by Schur.



            Indeed, we need to prove that
            $$sum_{cyc}(a-1)^3+frac{3}{4}geq0$$ or
            $$sum_{cyc}left(a^3-3a^2+3-1+frac{1}{4}right)geq0$$ or
            $$sum_{cyc}(a^3-(a+b+c)a^2)+frac{27}{4}$$ or
            $$4sum_{cyc}(-a^2b-a^2c)+(a+b+c)^3geq0$$ or
            $$sum_{cyc}(-4a^2b-4a^2c+a^3+3a^2b+3a^2c+2abc)geq0$$ or
            $$sum_{cyc}(a^3-a^2b-a^2c+2abc)geq0,$$ which is obvious.






            share|cite|improve this answer











            $endgroup$













            • $begingroup$
              Could you please explain more?
              $endgroup$
              – Lê Thành Đạt
              Mar 18 at 15:14










            • $begingroup$
              Yes, of course! I am driving now. I explain it later
              $endgroup$
              – Michael Rozenberg
              Mar 18 at 15:35










            • $begingroup$
              I can wait. Drive carefully!
              $endgroup$
              – Lê Thành Đạt
              Mar 18 at 15:37










            • $begingroup$
              It is just $9(a-1)^3+9(b-1)^3+9(c-1)^3ge -frac{left(a+b+cright)^3}{4}$.
              $endgroup$
              – Word Shallow
              Mar 18 at 16:35
















            0












            $begingroup$

            The homogenisation gives
            $$sum_{cyc}(a^3-a^2b-a^2c+2abc)geq0,$$
            which is true by Schur.



            Indeed, we need to prove that
            $$sum_{cyc}(a-1)^3+frac{3}{4}geq0$$ or
            $$sum_{cyc}left(a^3-3a^2+3-1+frac{1}{4}right)geq0$$ or
            $$sum_{cyc}(a^3-(a+b+c)a^2)+frac{27}{4}$$ or
            $$4sum_{cyc}(-a^2b-a^2c)+(a+b+c)^3geq0$$ or
            $$sum_{cyc}(-4a^2b-4a^2c+a^3+3a^2b+3a^2c+2abc)geq0$$ or
            $$sum_{cyc}(a^3-a^2b-a^2c+2abc)geq0,$$ which is obvious.






            share|cite|improve this answer











            $endgroup$













            • $begingroup$
              Could you please explain more?
              $endgroup$
              – Lê Thành Đạt
              Mar 18 at 15:14










            • $begingroup$
              Yes, of course! I am driving now. I explain it later
              $endgroup$
              – Michael Rozenberg
              Mar 18 at 15:35










            • $begingroup$
              I can wait. Drive carefully!
              $endgroup$
              – Lê Thành Đạt
              Mar 18 at 15:37










            • $begingroup$
              It is just $9(a-1)^3+9(b-1)^3+9(c-1)^3ge -frac{left(a+b+cright)^3}{4}$.
              $endgroup$
              – Word Shallow
              Mar 18 at 16:35














            0












            0








            0





            $begingroup$

            The homogenisation gives
            $$sum_{cyc}(a^3-a^2b-a^2c+2abc)geq0,$$
            which is true by Schur.



            Indeed, we need to prove that
            $$sum_{cyc}(a-1)^3+frac{3}{4}geq0$$ or
            $$sum_{cyc}left(a^3-3a^2+3-1+frac{1}{4}right)geq0$$ or
            $$sum_{cyc}(a^3-(a+b+c)a^2)+frac{27}{4}$$ or
            $$4sum_{cyc}(-a^2b-a^2c)+(a+b+c)^3geq0$$ or
            $$sum_{cyc}(-4a^2b-4a^2c+a^3+3a^2b+3a^2c+2abc)geq0$$ or
            $$sum_{cyc}(a^3-a^2b-a^2c+2abc)geq0,$$ which is obvious.






            share|cite|improve this answer











            $endgroup$



            The homogenisation gives
            $$sum_{cyc}(a^3-a^2b-a^2c+2abc)geq0,$$
            which is true by Schur.



            Indeed, we need to prove that
            $$sum_{cyc}(a-1)^3+frac{3}{4}geq0$$ or
            $$sum_{cyc}left(a^3-3a^2+3-1+frac{1}{4}right)geq0$$ or
            $$sum_{cyc}(a^3-(a+b+c)a^2)+frac{27}{4}$$ or
            $$4sum_{cyc}(-a^2b-a^2c)+(a+b+c)^3geq0$$ or
            $$sum_{cyc}(-4a^2b-4a^2c+a^3+3a^2b+3a^2c+2abc)geq0$$ or
            $$sum_{cyc}(a^3-a^2b-a^2c+2abc)geq0,$$ which is obvious.







            share|cite|improve this answer














            share|cite|improve this answer



            share|cite|improve this answer








            edited Mar 18 at 17:46

























            answered Mar 18 at 15:13









            Michael RozenbergMichael Rozenberg

            109k1896201




            109k1896201












            • $begingroup$
              Could you please explain more?
              $endgroup$
              – Lê Thành Đạt
              Mar 18 at 15:14










            • $begingroup$
              Yes, of course! I am driving now. I explain it later
              $endgroup$
              – Michael Rozenberg
              Mar 18 at 15:35










            • $begingroup$
              I can wait. Drive carefully!
              $endgroup$
              – Lê Thành Đạt
              Mar 18 at 15:37










            • $begingroup$
              It is just $9(a-1)^3+9(b-1)^3+9(c-1)^3ge -frac{left(a+b+cright)^3}{4}$.
              $endgroup$
              – Word Shallow
              Mar 18 at 16:35


















            • $begingroup$
              Could you please explain more?
              $endgroup$
              – Lê Thành Đạt
              Mar 18 at 15:14










            • $begingroup$
              Yes, of course! I am driving now. I explain it later
              $endgroup$
              – Michael Rozenberg
              Mar 18 at 15:35










            • $begingroup$
              I can wait. Drive carefully!
              $endgroup$
              – Lê Thành Đạt
              Mar 18 at 15:37










            • $begingroup$
              It is just $9(a-1)^3+9(b-1)^3+9(c-1)^3ge -frac{left(a+b+cright)^3}{4}$.
              $endgroup$
              – Word Shallow
              Mar 18 at 16:35
















            $begingroup$
            Could you please explain more?
            $endgroup$
            – Lê Thành Đạt
            Mar 18 at 15:14




            $begingroup$
            Could you please explain more?
            $endgroup$
            – Lê Thành Đạt
            Mar 18 at 15:14












            $begingroup$
            Yes, of course! I am driving now. I explain it later
            $endgroup$
            – Michael Rozenberg
            Mar 18 at 15:35




            $begingroup$
            Yes, of course! I am driving now. I explain it later
            $endgroup$
            – Michael Rozenberg
            Mar 18 at 15:35












            $begingroup$
            I can wait. Drive carefully!
            $endgroup$
            – Lê Thành Đạt
            Mar 18 at 15:37




            $begingroup$
            I can wait. Drive carefully!
            $endgroup$
            – Lê Thành Đạt
            Mar 18 at 15:37












            $begingroup$
            It is just $9(a-1)^3+9(b-1)^3+9(c-1)^3ge -frac{left(a+b+cright)^3}{4}$.
            $endgroup$
            – Word Shallow
            Mar 18 at 16:35




            $begingroup$
            It is just $9(a-1)^3+9(b-1)^3+9(c-1)^3ge -frac{left(a+b+cright)^3}{4}$.
            $endgroup$
            – Word Shallow
            Mar 18 at 16:35


















            draft saved

            draft discarded




















































            Thanks for contributing an answer to Mathematics Stack Exchange!


            • Please be sure to answer the question. Provide details and share your research!

            But avoid



            • Asking for help, clarification, or responding to other answers.

            • Making statements based on opinion; back them up with references or personal experience.


            Use MathJax to format equations. MathJax reference.


            To learn more, see our tips on writing great answers.




            draft saved


            draft discarded














            StackExchange.ready(
            function () {
            StackExchange.openid.initPostLogin('.new-post-login', 'https%3a%2f%2fmath.stackexchange.com%2fquestions%2f3152853%2fprove-that-a-13-b-13-c-13-ge-dfrac34-where-abc-3%23new-answer', 'question_page');
            }
            );

            Post as a guest















            Required, but never shown





















































            Required, but never shown














            Required, but never shown












            Required, but never shown







            Required, but never shown

































            Required, but never shown














            Required, but never shown












            Required, but never shown







            Required, but never shown







            Popular posts from this blog

            Magento 2 - Add success message with knockout Planned maintenance scheduled April 23, 2019 at 23:30 UTC (7:30pm US/Eastern) Announcing the arrival of Valued Associate #679: Cesar Manara Unicorn Meta Zoo #1: Why another podcast?Success / Error message on ajax request$.widget is not a function when loading a homepage after add custom jQuery on custom themeHow can bind jQuery to current document in Magento 2 When template load by ajaxRedirect page using plugin in Magento 2Magento 2 - Update quantity and totals of cart page without page reload?Magento 2: Quote data not loaded on knockout checkoutMagento 2 : I need to change add to cart success message after adding product into cart through pluginMagento 2.2.5 How to add additional products to cart from new checkout step?Magento 2 Add error/success message with knockoutCan't validate Post Code on checkout page

            Fil:Tokke komm.svg

            Where did Arya get these scars? Unicorn Meta Zoo #1: Why another podcast? Announcing the arrival of Valued Associate #679: Cesar Manara Favourite questions and answers from the 1st quarter of 2019Why did Arya refuse to end it?Has the pronunciation of Arya Stark's name changed?Has Arya forgiven people?Why did Arya Stark lose her vision?Why can Arya still use the faces?Has the Narrow Sea become narrower?Does Arya Stark know how to make poisons outside of the House of Black and White?Why did Nymeria leave Arya?Why did Arya not kill the Lannister soldiers she encountered in the Riverlands?What is the current canonical age of Sansa, Bran and Arya Stark?